2014 dxdy logo

Научный форум dxdy

Математика, Физика, Computer Science, Machine Learning, LaTeX, Механика и Техника, Химия,
Биология и Медицина, Экономика и Финансовая Математика, Гуманитарные науки




Начать новую тему Ответить на тему
 
 Иррациональные неравенства от четырёх переменных
Сообщение17.01.2021, 15:55 
Заслуженный участник


26/06/07
1929
Tel-aviv
Пусть $a,$ $b$, $c$ и $d$ неотрицательные числа, для которых $a+b+c+d=4$. Докажите, что:
1. $$\sqrt{\frac{a}{b+1}}+\sqrt{\frac{b}{c+1}}+\sqrt{\frac{c}{d+1}}+\sqrt{\frac{d}{a+1}}\leq3;$$
2. $$\sqrt{\frac{a}{b+3}}+\sqrt{\frac{b}{c+3}}+\sqrt{\frac{c}{d+3}}+\sqrt{\frac{d}{a+3}}\leq2;$$
3. $$\sqrt[4]{\frac{a}{10b+1}}+\sqrt[4]{\frac{b}{10c+1}}+\sqrt[4]{\frac{c}{10d+1}}+\sqrt[4]{\frac{d}{10a+1}}\leq6\sqrt[4]{\frac{2}{55}};$$
4. $$\sqrt[5]{\frac{a}{5b+1}}+\sqrt[5]{\frac{b}{5c+1}}+\sqrt[5]{\frac{c}{5d+1}}+\sqrt[5]{\frac{d}{5a+1}}\leq\sqrt[5]{200};$$
5. Пусть $a,$ $b$, $c$ и $d$ неотрицательные числа, для которых $a+b+c+d=5$. Докажите, что:
$$\sqrt[3]{\frac{a}{b+1}}+\sqrt[3]{\frac{b}{c+1}}+\sqrt[3]{\frac{c}{d+1}}+\sqrt[3]{\frac{d}{a+1}}\leq\sqrt[3]{36}.$$

(PS)

Все оценки точные. Первая и пятая задачи имеют красивое решение в одну строчку. Вторая также имеет простое решение, но не в одну строчку.

 Профиль  
                  
 
 Re: Иррациональные неравенства от четырёх переменных
Сообщение18.01.2021, 08:29 
Заблокирован


16/04/18

1129
Если бы у бабушки... В смысле если бы в последней задаче сумма была бы не 5, а 4, то она следовала бы из первой и неравенства о средних.

 Профиль  
                  
 
 Re: Иррациональные неравенства от четырёх переменных
Сообщение18.01.2021, 11:23 
Заблокирован


16/04/18

1129
Для 1) пока получилось простыми средствами с худшей постоянной справа=4.

 Профиль  
                  
 
 Re: Иррациональные неравенства от четырёх переменных
Сообщение04.02.2021, 06:19 
Модератор
Аватара пользователя


11/01/06
5660
arqady, напомнило вот эти ваши неравенства:
arqady в сообщении #106301 писал(а):
Пусть $a,$ $b$ и $c$ неотрицательные числа, никакие два из которых не равны нулю. Докажите, что
1) $$\sqrt{1+\frac{48a}{b+c}}+\sqrt{1+\frac{48b}{a+c}}+\sqrt{1+\frac{48c}{a+b}}\geq15.$$
2)$$\sqrt{\frac{a}{4a+5b}}+\sqrt{\frac{b}{4b+5c}}+\sqrt{\frac{c}{4c+5a}}\leq1.$$
3)$$\sqrt[3]{\frac{a}{2b+25c}}+\sqrt[3]{\frac{b}{2c+25a}}+\sqrt[3]{\frac{c}{2a+25b}}\geq1.$$
Имеется мощный совершенно элементарный общий метод доказательства. Сам нашёл! :D

Неравенства из этой темы тем же общим методом доказываются?

 Профиль  
                  
 
 Re: Иррациональные неравенства от четырёх переменных
Сообщение04.02.2021, 11:28 
Заслуженный участник


26/06/07
1929
Tel-aviv
Нет. Эти (первое и второе принадлежат Василе Кыртоаже) неравенства доказываются от противного(могу показать, как конкрентно). Уже для третьего неравенства это очень трудно реализовать.
Но можно воспользоваться ещё теоремой о право выпуклой функции Василе Кыртоаже. Мои неравенства доказываются другим способом.
Кстати, недавно (в этот понедельник) выяснилось, что и следующее неравенство можно доказать в одну строчку.
Пусть $a,$ $b$, $c$ и $d$ неотрицательные числа, для которых $a+b+c+d=4$. Докажите, что:
$$\sqrt{\frac{a}{b+2}}+\sqrt{\frac{b}{c+2}}+\sqrt{\frac{c}{d+2}}+\sqrt{\frac{d}{a+2}}\leq\frac{4}{\sqrt3}$$

 Профиль  
                  
 
 Re: Иррациональные неравенства от четырёх переменных
Сообщение12.02.2021, 19:26 
Заслуженный участник


03/01/09
1677
москва
В 1) получилось доказать лишь более слабое неравенство.
1) Применим к первому и третьему, а также ко второму и четвертому слагаемым суммы $S$ неравенство К.-Б., получим:$$S\leq \sqrt {(a+c)(\frac 1{b+1}+\frac 1{d+1})}+\sqrt {(b+d)(\frac 1{a+1}+\frac 1{c+1})}$$Обозначим $a+c=x, b+d=4-x, \dfrac 1{a+1}+\dfrac 1{c+1}=\dfrac {x+2}{ac+x+1}\leq \dfrac {x+2}{x+1}$. Аналогично получим: $\dfrac 1{b+1}+\dfrac 1{d+1}\leq \dfrac {6-x}{5-x}$
Таким образом:$$S\leq \sqrt {\frac {x(6-x)}{5-x}}+\sqrt {\frac {(4-x)(x+2)}{x+1}}\eqno (1)$$
Правая часть неравенства (1) достигает наибольшего значения при $x=2$, поэтому $S\leq 4\sqrt {\frac 23}\approx 3.3$
Таким же способом можно получить оценку для суммы 2) и вообще для сумм вида $S_n=\sqrt {\dfrac a{b+n}}+\cdots$, где $a+b+c+d=4$.
В связи с этим можно предположить, что в общем случае выполняется неравенство: $S_n\leq \dfrac 4{\sqrt {n+1}}$,(это значение достигается при $a=b=c=d=1$,и к нему асимптотически приближается верхняя оценка для $S_n$) т.е., например, неравенство 1) можно было бы еще улучшить до $S\leq 2\sqrt 2$.

 Профиль  
                  
 
 Re: Иррациональные неравенства от четырёх переменных
Сообщение12.02.2021, 23:09 
Аватара пользователя


26/02/14
497
so dna
mihiv в сообщении #1504873 писал(а):
В связи с этим можно предположить, что в общем случае выполняется неравенство: $S_n\leq \dfrac 4{\sqrt {n+1}}$,(это значение достигается при $a=b=c=d=1$,и к нему асимптотически приближается верхняя оценка для $S_n$) т.е., например, неравенство 1) можно было бы еще улучшить до $S\leq 2\sqrt 2$.
Это неверно, т.к. равенство в 1) достигается при $a=c=1+\frac{2\sqrt{5}}{5}, b=d=1-\frac{2\sqrt{5}}{5}$ и его нельзя улучшить путём уменьшения правой части на константу.

 Профиль  
                  
 
 Re: Иррациональные неравенства от четырёх переменных
Сообщение12.02.2021, 23:43 
Заслуженный участник


03/01/09
1677
москва
Rak so dna
Меня ввело в заблуждение то, что для $n=2,3$ неравенства выполняются.

 Профиль  
                  
 
 Re: Иррациональные неравенства от четырёх переменных
Сообщение13.02.2021, 17:30 
Заблокирован


16/04/18

1129
Вопрос: это задача на применение не общеизвестной специальной теоремы про выпуклые/вогнутые функции? Или есть решение без этого, в мейнстриме?

 Профиль  
                  
 
 Re: Иррациональные неравенства от четырёх переменных
Сообщение13.02.2021, 18:48 
Заслуженный участник


26/06/07
1929
Tel-aviv
В моём однострочечном решении я пользовался AM-GM и Jensen.

 Профиль  
                  
 
 Re: Иррациональные неравенства от четырёх переменных
Сообщение16.04.2021, 21:31 
Аватара пользователя


26/02/14
497
so dna
arqady в сообщении #106301 писал(а):
Пусть $a,$ $b$ и $c$ неотрицательные числа, никакие два из которых не равны нулю. Докажите, что
1) $$\sqrt{1+\frac{48a}{b+c}}+\sqrt{1+\frac{48b}{a+c}}+\sqrt{1+\frac{48c}{a+b}}\geq15.$$
Обозначим $\frac{2a}{b+c}=A, \frac{2b}{c+a}=B, \frac{2c}{a+b}=C,$ тогда достаточно доказать $\sum\limits_{cyc}\sqrt{24A+1}\geqslant15,$ для неотрицательных $A,B,C$ таких, что $\sum\limits_{cyc}\frac{1}{A+2}=1.$
От противного: предположим, что существуют такие $A_0,B_0,C_0\geqslant0, \quad \sum\limits_{cyc}\frac{1}{A_0+2}=1,$ что $\sum\limits_{cyc}\sqrt{24A_0+1}<15.$ Тогда, существует такое $\theta>1$, при котором $\sum\limits_{cyc}\sqrt{24\theta A_0+1}=15.$ Обозначим $A_1=\theta A_0, B_1=\theta B_0, C_1=\theta C_0,$ тогда имеем: $\sum\limits_{cyc}\sqrt{24A_1+1}=15; \quad \sum\limits_{cyc}\frac{1}{A_1+2} = \sum\limits_{cyc}\frac{1}{\theta A_0+2} < 1; \quad A_1,B_1,C_1\geqslant0.$ Теперь, поскольку $\sum\limits_{cyc}\frac{1}{A_1+2} < 1 \Leftrightarrow A_1B_1C_1+A_1B_1+B_1C_1+C_1A_1 > 4,$ для получения противоречия достаточно доказать, что для любых $X,Y,Z\geqslant0,$ для которых $\sum\limits_{cyc}\sqrt{24X+1}=15,$ выполняется неравенство $XYZ+XY+YZ+ZX \leqslant 4.$ Докажем это:
Для любых $X,Y,Z\geqslant0,$ найдутся такие $x,y,z\geqslant0,$ что $X=\frac{x^2+x}{6}, Y=\frac{y^2+y}{6}, Z=\frac{z^2+z}{6}$, поэтому достаточно доказать для неотрицательных $x,y,z,$ таких, что $x+y+z=6$ неравенство $$f(x,y,z)=4-\frac{1}{6^3}xyz(x+1)(y+1)(z+1)-\frac{1}{6^2}\sum\limits_{cyc}xy(x+1)(y+1)\geqslant0$$Имеем:
$f(x,y,z)\stackrel{AM-GM}{\geqslant}4-\frac{1}{6^3}xyz\left(\frac{x+y+z+3}{3}\right)^3-\frac{1}{6^2}\sum\limits_{cyc}xy(x+1)(y+1)=$
$=4\left(\frac{x+y+z}{6}\right)^4-\frac{1}{8}xyz\left(\frac{x+y+z}{6}\right)-\frac{1}{6^2}\sum\limits_{cyc}xy\left(x+\left(\frac{x+y+z}{6}\right)\right)\left(y+\left(\frac{x+y+z}{6}\right)\right)=$
$=\frac{1}{6^4}\sum\limits_{cyc}(x-y)^2(13xy+2(x+y-z)^2) \geqslant0.$

arqady в сообщении #106301 писал(а):
Пусть $a,$ $b$ и $c$ неотрицательные числа, никакие два из которых не равны нулю. Докажите, что
2)$$\sqrt{\frac{a}{4a+5b}}+\sqrt{\frac{b}{4b+5c}}+\sqrt{\frac{c}{4c+5a}}\leq1.$$

$\Leftrightarrow \sum\limits_{cyc}\sqrt{\frac{1}{4+5\frac{b}{a}}} \leqslant1  \Leftrightarrow A=\frac{b}{a}, B = \frac{c}{b}, C=\frac{a}{c}, \quad \sum\limits_{cyc}\sqrt{\frac{1}{4+5A}}\leqslant1$
$ABC = 1$ и достаточно доказать это для $A,B,C\geqslant0$
От противного: предположим, что существуют такие $A_0,B_0,C_0\geqslant0, \quad A_0B_0C_0=1,$ что $\sum\limits_{cyc}\sqrt{\frac{1}{4+5A_0}}>1.$ Тогда, существует такое $\theta > 1$, при котором $\sum\limits_{cyc}\sqrt{\frac{1}{4+5\theta A_0}}=1.$ Обозначим $A_1=\theta A_0, B_1=\theta B_0, C_1=\theta C_0,$ тогда имеем: $\sum\limits_{cyc}\sqrt{\frac{1}{4+5A_1}}=1; \quad  A_1B_1C_1=\theta^3A_0B_0C_0>1; \quad A_1,B_1,C_1\geqslant0.$ Для получения противоречия достаточно доказать, что для любых $X,Y,Z\geqslant0,$ для которых $\sum\limits_{cyc}\sqrt{\frac{1}{4+5X}}=1,$ выполняется неравенство $XYZ \leqslant 1.$
Докажем это:
Для любых $X,Y,Z\geqslant0,$ найдутся такие $p,q,r\geqslant0,$ что $X=\frac{p^2+4p}{5}, Y=\frac{q^2+4q}{5}, Z=\frac{r^2+4r}{5}$, поэтому достаточно доказать для неотрицательных $p,q,r,$ таких, что $\sum\limits_{cyc}\frac{1}{p+2}=1$ неравенство $\prod\limits_{cyc}p(p+4)\leqslant5^3.$
$\Leftrightarrow \prod\limits_{cyc}\frac{2x}{y+z}\left(\frac{2x}{y+z}+4\right)\leqslant5^3; \quad x,y,z\geqslant0 $

(почему неотрицательные?)

покажем, что $x,y,z$ можно считать неотрицательными. Имеем:
$$\begin{cases}
p=\frac{2x}{y+z}\geqslant0\\
q=\frac{2y}{z+x}\geqslant0\\
r=\frac{2z}{x+y}\geqslant0
\end{cases} \Rightarrow 
\begin{cases}
x(y+z)\geqslant0\\
y(z+x)\geqslant0\\
z(x+y)\geqslant0
\end{cases} \Rightarrow 
\begin{cases}
xy=\frac{x^2y^2z^2+x^2y^2\big(y(z+x)+x(y+z)\big)}{x^2y^2+y(z+x)\cdot x(y+z)}\geqslant0\\
yz=\frac{x^2y^2z^2+y^2z^2\big(z(x+y)+y(z+x)\big)}{y^2z^2+z(x+y)\cdot y(z+x)}\geqslant0\\
zx=\frac{x^2y^2z^2+z^2x^2\big(x(y+z)+z(x+y)\big)}{z^2x^2+x(y+z)\cdot z(x+y)}\geqslant0\\
\end{cases} 
$$ поэтому $x,y,z$ одного знака, но $-x,-y,-z$ соответствуют $x,y,z$ а значит мы имеем право считать их неотрицательными

$\Leftrightarrow \sum\limits_{cyc}z(x-y)^2(125z^3+125z^2(x+y)+101xyz+122xy(x+y))\geqslant0$

arqady в сообщении #106301 писал(а):
Пусть $a,$ $b$ и $c$ неотрицательные числа, никакие два из которых не равны нулю. Докажите, что
3)$$\sqrt[3]{\frac{a}{2b+25c}}+\sqrt[3]{\frac{b}{2c+25a}}+\sqrt[3]{\frac{c}{2a+25b}}\geq1.$$

$\Leftrightarrow \sum\limits_{cyc}\sqrt[3]{\frac{1}{2\frac{b}{c}\frac{c}{a}+25\frac{c}{a}}} \geqslant1 \Leftrightarrow A=\frac{a}{b}, B = \frac{b}{c}, C=\frac{c}{a}, \quad \sum\limits_{cyc}\sqrt[3]{\frac{1}{2BC+25C}}\geqslant1$
$ABC = 1; \quad A,B,C>0$
От противного: предположим, что существуют такие $A_0,B_0,C_0>0, \quad A_0B_0C_0=1,$ что $\sum\limits_{cyc}\sqrt[3]{\frac{1}{2B_0C_0+25C_0}}<1.$ Тогда, существует такое $0<\theta < 1$, при котором $\sum\limits_{cyc}\sqrt[3]{\frac{1}{2\theta^2B_0C_0+25\theta C_0}}=1.$
Обозначим $A_1=\theta A_0, \quad B_1=\theta B_0, \quad C_1=\theta C_0,$ тогда имеем: $\sum\limits_{cyc}\sqrt[3]{\frac{1}{2B_1C_1+25C_1}}=1; \quad  A_1B_1C_1=\theta^3A_0B_0C_0<1; \quad A_1,B_1,C_1>0.$ Для получения противоречия достаточно доказать, что для любых $X,Y,Z>0,$ для которых $\sum\limits_{cyc}\sqrt[3]{\frac{1}{2YZ+25Z}}=1,$ выполняется неравенство $XYZ \geqslant 1.$
Докажем это:
Пусть $X(2Z+25)=\frac{1}{x^3}, \quad Y(2X+25)=\frac{1}{y^3}, \quad Z(2Y+25)=\frac{1}{z^3}; \quad x,y,z>0$ Тогда

$2(XY+YZ+ZX)+25(X+Y+Z)=\frac{1}{x^3}+\frac{1}{y^3}+\frac{1}{z^3}$ и

$XYZ(2X+25)(2Y+25)(2Z+25)=$

$=8(XYZ)^2+XYZ\big(50(2(XY+YZ+ZX)+25(X+Y+Z))+25^3\big)=$

$=8(XYZ)^2+XYZ\left(50\left(\frac{1}{x^3}+\frac{1}{y^3}+\frac{1}{z^3}\right)+25^3\right)=\frac{1}{x^3y^3z^3},$ отсюда

$XYZ=\frac{1}{16}\sqrt{625\left(2\left(\frac{1}{x^3}+\frac{1}{y^3}+\frac{1}{z^3}\right)+625\right)^2+\frac{32}{x^3y^3z^3}}-\frac{25}{16}\left(2\left(\frac{1}{x^3}+\frac{1}{y^3}+\frac{1}{z^3}\right)+625\right)$
Поэтому для $x+y+z=1$ надо доказать, что
$$\frac{1}{16}\sqrt{625\left(2\left(\frac{1}{x^3}+\frac{1}{y^3}+\frac{1}{z^3}\right)+625\right)^2+\frac{32}{x^3y^3z^3}}\geqslant\frac{25}{16}\left(2\left(\frac{1}{x^3}+\frac{1}{y^3}+\frac{1}{z^3}\right)+625\right)+1$$

$\Leftrightarrow 1-50(x^3y^3+y^3z^3+z^3x^3)-15633x^3y^3z^3\geqslant0, \quad x+y+z=1$

$\Leftrightarrow (x+y+z)^9-50(x+y+z)^3(x^3y^3+y^3z^3+z^3x^3)-15633x^3y^3z^3\geqslant0$

(SOS)

$LHS = \frac{1}{2} \sum\limits_{cyc}(y-z)^2\big(966x^4yz(y+z)+4701x^3y^2z^2+\\(111(y^5+z^5)+870yz(y^3+z^3)+2385y^2z^2(y+z))x^2+(72yz(y^4+z^4)+636y^3z^3)x+y^7+z^7+20yz(y^5+z^5)+48y^3z^3(y+z)\big)\geqslant0$

Можно ещё BW

 Профиль  
                  
 
 Re: Иррациональные неравенства от четырёх переменных
Сообщение30.05.2021, 10:38 
Аватара пользователя


26/02/14
497
so dna
arqady в сообщении #1501573 писал(а):
Пусть $a,$ $b$, $c$ и $d$ неотрицательные числа, для которых $a+b+c+d=4$. Докажите, что:...
Скорее всего верно следующее:

Определим многочлены $P_{n}(t)$ и $Q_{n}(t)$ из разложения
$2^{n-1}t^{\left[\frac{n+1}{2}\right]}P_{n}(t)+2^{n-1}t^{\left[\frac{n}{2}\right]}Q_{n}(t)\sqrt{t(t+4)} = \left(t+\sqrt{t(t+4)}\right)^n,$
тогда для неотрицательных $k,$ $a,$ $b,$ $c$ и $d,$ для которых $a+b+c+d=S$ выполняется:
____

$1.\quad m=2n:$
$$\sqrt[m]{\frac{a}{b+k}}+\sqrt[m]{\frac{b}{c+k}}+\sqrt[m]{\frac{c}{d+k}}+\sqrt[m]{\frac{d}{a+k}}\leq\begin{cases}
2(\gamma+2)\sqrt[m]{\frac{P_{2n-1}(\gamma)}{P_{2n+1}(\gamma)}},\quad S\geqslant 2k(2n-1)\\
4\sqrt[m]{\frac{S}{S+4k}},\quad S\leqslant k(2n-1)
\end{cases}$$ где $\gamma$ - неотрицательный корень уравнения $P_{2n-1}(\gamma)Q_{2n+1}(\gamma)=\frac{S}{2k},$
____

$2.\quad m=4n-1:$
$$\sqrt[m]{\frac{a}{b+k}}+\sqrt[m]{\frac{b}{c+k}}+\sqrt[m]{\frac{c}{d+k}}+\sqrt[m]{\frac{d}{a+k}}\leq\begin{cases}
2\sqrt[m]{\frac{P_{2n-1}(\gamma)}{Q_{2n}(\gamma)}(\gamma+4)^{2n-1}},\quad S\geqslant 4k(2n-1)\\
4\sqrt[m]{\frac{S}{S+4k}},\quad S\leqslant 2k(2n-1)
\end{cases}$$ где $\gamma$ - неотрицательный корень уравнения $P_{2n-1}(\gamma)P_{2n}(\gamma)=\frac{S}{2k},$
____

$3.\quad m=4n+1:$
$$\sqrt[m]{\frac{a}{b+k}}+\sqrt[m]{\frac{b}{c+k}}+\sqrt[m]{\frac{c}{d+k}}+\sqrt[m]{\frac{d}{a+k}}\leq\begin{cases}
2\sqrt[m]{\frac{Q_{2n}(\gamma)}{P_{2n+1}(\gamma)}(\gamma+4)^{2n+1}},\quad S\geqslant 8kn\\
4\sqrt[m]{\frac{S}{S+4k}},\quad S\leqslant 4kn
\end{cases}$$ где $\gamma$ - неотрицательный корень уравнения $(\gamma+4)Q_{2n}(\gamma)Q_{2n+1}(\gamma)=\frac{S}{2k},$
_______________________________________

$1.\quad m=2n$ тезисное рассуждение:
Рассмотрим функцию $F(x, s) =\sqrt[m]{\frac{x}{s-x+1}}+\sqrt[m]{\frac{s-x}{x+1}}$ на $x,s\geqslant0$

$F'_x\geqslant0 \Leftrightarrow G(x)=(x-s)^{2n-1}(x+1)^{2n+1}-x^{2n-1}(x-s-1)^{2n+1}\leqslant0$

$G(x)=(x-\frac{s}{2})(x-x_1)(x-x_2)R(x)$ где

$x_{1,2}=\frac{s}{2}\pm\frac{P_{2n-1}(\alpha)P_{2n+1}(\alpha)}{2}\sqrt{\frac{\alpha}{\alpha+4}}\geqslant0$

$\alpha$ - неотрицательный корень уравнения $P_{2n-1}(\alpha)Q_{2n+1}(\alpha)=s\Rightarrow s\geqslant 2n-1$

$R(x)$

(не имеет положительных корней)

для $x\geqslant s+1$ проверяем положительность коэффициентов многочлена $G(s+1+t)$
для $s\leqslant x\leqslant s+1$ проверяем положительность коэффициентов многочлена $G(s+\frac{1}{1+t})$
для $0\leqslant x\leqslant s$ проверяем положительность коэффициентов многочленов $R(\frac{s}{2}\pm\frac{\sqrt{s^2-4t}}{2})$

Поэтому уравнение $G(x)=0$ имеет три неотрицательных корня, при $s\geqslant 2n-1$ и один неотрицательный корень при $s\leqslant 2n-1$

Поэтому $$F_{\max}(s)=\begin{cases}
F(x_1,s)=F(x_2,s)=(\alpha+2)\sqrt[m]{\frac{P_{2n-1}(\alpha)}{P_{2n+1}(\alpha)}},\quad s\geqslant2n-1\\
F(\frac{s}{2},s)=2\sqrt[m]{\frac{s}{s+2}},\quad s\leqslant 2n-1
\end{cases}$$
Т.о. $\sqrt[m]{\frac{x}{w+k}}+\sqrt[m]{\frac{w}{x+k}}=F(\frac{x}{k}, \frac{x+w}{k})\leqslant F_{\max}(\frac{x+w}{k})=\begin{cases}
(\alpha+2)\sqrt[m]{\frac{P_{2n-1}(\alpha)}{P_{2n+1}(\alpha)}},\quad x+w\geqslant k(2n-1)\\
2\sqrt[m]{\frac{x+w}{x+w+2k}},\quad x+w\leqslant k(2n-1)
\end{cases}$

где $\alpha$ - неотрицательный корень уравнения $P_{2n-1}(\alpha)Q_{2n+1}(\alpha)=\frac{x+w}{k},$
____

Отметим для всех $t\geqslant 0$:

Утверждение $T_1:$ функция $(\theta_t+2)\sqrt[m]{\frac{P_{2n-1}(\theta_t)}{P_{2n+1}(\theta_t)}}$ является вогнутой

Утверждение $T_2:\quad (\theta_t+2)\sqrt[m]{\frac{P_{2n-1}(\theta_t)}{P_{2n+1}(\theta_t)}}\geqslant F(\frac{t}{2},t)$

тут $\theta_t$ - наибольший (не обязательно неотрицательный) корень уравнения $P_{2n-1}(\theta_t)Q_{2n+1}(\theta_t)=t$
____

Пусть $x,y,z,w$ - это числа $a,b,c,d$ расположенные по возрастанию, тогда
$\sqrt[m]{x}\leqslant\sqrt[m]{y}\leqslant\sqrt[m]{z}\leqslant\sqrt[m]{w}$
$\sqrt[m]{\frac{1}{w+k}}\leqslant\sqrt[m]{\frac{1}{z+k}}\leqslant\sqrt[2n]{\frac{1}{y+k}}\leqslant\sqrt[m]{\frac{1}{x+k}}$ и по перестановочному неравенству

$\sum\limits_{cyc}\sqrt[m]{\frac{a}{b+k}}\leqslant\left(\sqrt[m]{\frac{x}{w+k}}+\sqrt[m]{\frac{w}{x+k}}\right)+\left(\sqrt[2n]{\frac{y}{z+k}}+\sqrt[m]{\frac{z}{y+k}}\right)\leqslant$
$\leqslant F_{\max}(\frac{x+w}{k})+F_{\max}(\frac{y+z}{k})$

Случай $S\leqslant k(2n-1):$
$\begin{cases}
x+w\leqslant S\leqslant k(2n-1)\\
y+z\leqslant S\leqslant k(2n-1)
\end{cases}\Rightarrow F_{\max}(\frac{x+w}{k})+F_{\max}(\frac{y+z}{k})=$
$= 2\sqrt[m]{\frac{x+w}{x+w+2k}}+2\sqrt[m]{\frac{y+z}{y+z+2k}}\stackrel{Jensen}\leqslant 4\sqrt[m]{\frac{\frac{x+y+z+w}{2}}{\frac{x+y+z+w}{2}+2k}}=4\sqrt[m]{\frac{S}{S+4k}}$

Случай $S\geqslant 2k(2n-1):$

$F_{\max}(\frac{x+w}{k})+F_{\max}(\frac{y+z}{k})\stackrel{T_2}\leqslant$

$(\theta_{(x+w)}+2)\sqrt[m]{\frac{P_{2n-1}(\theta_{(x+w)})}{P_{2n+1}(\theta_{(x+w)})}}+(\theta_{(y+z)}+2)\sqrt[m]{\frac{P_{2n-1}(\theta_{(y+z)})}{P_{2n+1}(\theta_{(y+z)})}}\stackrel{T_1\; and\; Jensen}\leqslant $

$(\gamma+2)\sqrt[m]{\frac{P_{2n-1}(\gamma)}{P_{2n+1}(\gamma)}}$

где $\theta_t$ - наибольший (не обязательно неотрицательный) корень уравнения $P_{2n-1}(\theta_t)Q_{2n+1}(\theta_t)=\frac{t}{k},$
$\gamma$ - неотрицательный корень уравнения
$P_{2n-1}(\gamma)Q_{2n+1}(\gamma)=\frac{S}{2k}$ (условие $S\geqslant 2k(2n-1)$ гарантирует такой корень. При остальных $S\geqslant0$ эта оценка неравенства $1.$ сохраняется, но равенство уже не достигается).
_____________________

Оставшиеся случаи $m=4n\pm1$ рассматриваются, в общем аналогично, только уже для функции $F(\frac{s}{1+x^2},s),$ поскольку у $F(x,s)$ проблемы с неотрицательностью одного из корней.

 Профиль  
                  
 
 Re: Иррациональные неравенства от четырёх переменных
Сообщение27.12.2022, 16:31 
Аватара пользователя


26/02/14
497
so dna
arqady в сообщении #1501573 писал(а):
5. Пусть $a,$ $b$, $c$ и $d$ неотрицательные числа, для которых $a+b+c+d=5$. Докажите, что:
$$\sqrt[3]{\frac{a}{b+1}}+\sqrt[3]{\frac{b}{c+1}}+\sqrt[3]{\frac{c}{d+1}}+\sqrt[3]{\frac{d}{a+1}}\leq\sqrt[3]{36}.$$

Воспользуемся:

arqady в сообщении #456475 писал(а):
Пусть $a_i$, $b_i$ и $c_i$, $\alpha$, $\beta$ и $\gamma$ положительны. Тогда согласно Гёльдеру:
$(a_1+a_2+...+a_k)^{\alpha}(b_1+b_2+...+b_k)^{\beta}(c_1+c_2+...+c_k)^{\gamma}\geq\left(\sum\limits_{i=1}^k\left(a_i^{\alpha}b_i^{\beta}\right)^{\frac{1}{\alpha+\beta}}\right)^{\alpha+\beta}\left(\sum\limits_{i=1}^kc_i\right)^{\gamma}\geq$
$\geq\left(\sum\limits_{i=1}^k\left(a_i^{\alpha}b_i^{\beta}c_i^{\gamma}\right)^{\frac{1}{\alpha+\beta+\gamma}}\right)^{\alpha+\beta+\gamma}$.
Всё это аналогично продолжается на любое число последовательностей положительных чисел.

Имеем:

$\sum\limits_{cyc}{\sqrt[3]{\frac{a}{b+1}}}=
\sum\limits_{cyc}(a+1)^\frac13\left(\frac{a}{a+1}\right)^\frac13\left(\frac{1}{b+1} \right)^\frac13 \leq \left(\sum\limits_{cyc}(a+1)\right)^\frac13\left(\sum\limits_{cyc}\frac{a}{a+1}\right)^\frac13\left(\sum\limits_{cyc}\frac{1}{b+1} \right)^\frac13=$

$=\sqrt[3]{9\sum\limits_{cyc}\frac{a}{a+1}\sum\limits_{cyc}\frac{1}{a+1}}\leq \sqrt[3]{9\left(\frac{\sum\limits_{cyc}\frac{a}{a+1}+\sum\limits_{cyc}\frac{1}{a+1}}2 \right)^2}=\sqrt[3]{36}$

 Профиль  
                  
Показать сообщения за:  Поле сортировки  
Начать новую тему Ответить на тему  [ Сообщений: 13 ] 

Модераторы: Модераторы Математики, Супермодераторы



Кто сейчас на конференции

Сейчас этот форум просматривают: нет зарегистрированных пользователей


Вы не можете начинать темы
Вы не можете отвечать на сообщения
Вы не можете редактировать свои сообщения
Вы не можете удалять свои сообщения
Вы не можете добавлять вложения

Найти:
Powered by phpBB © 2000, 2002, 2005, 2007 phpBB Group